Difference between revisions of "2013 AIME I Problems/Problem 8"

(Solution)
(Solution)
Line 10: Line 10:
 
<math>\frac{1}{mn} \le x \le \frac{m}{n}</math>
 
<math>\frac{1}{mn} \le x \le \frac{m}{n}</math>
  
<math>\frac{1}{mn} - \frac{m}{n} = \frac{1}{2013}</math>
+
<math>\frac{m}{n} - \frac{1}{mn} = \frac{1}{2013}</math>
 +
 
 +
<math>n = 2013m - \frac{2013}{m}</math>
 +
 
 +
For ''n'' to be an integer, ''m'' must divide 2013, and ''m'' > 1. To minimize ''n'', ''m'' should be as small as possible because increasing ''m'' will decrease <math>\frac{2013}{m}</math> , the amount you are subtracting, and increase 2013''m'' , the amount you are adding; this also leads to a small ''m'' which clearly minimizes ''m'' + ''n''.
 +
 
 +
We let ''m'' equal 3, the smallest non-1 factor of 2013. <math>n = 2013*3 - \frac{2013}{3} = 6039 - 671 = 5368</math>
 +
 
 +
<math>m + n = 5371,</math> so the answer is <math>\boxed{371}</math>

Revision as of 21:12, 15 March 2013

Problem 8

The domain of the function f(x) = arcsin(log$_{m}$(nx)) is a closed interval of length $\frac{1}{2013}$ , where m and n are positive integers and m > 1. Find the remainder when the smallest possible sum m + n is divided by 1000.


Solution

The domain of the arcsin function is [-1, 1], so -1 $\le$ log$_{m}$(nx) $\le$ 1.

$\frac{1}{m} \le nx \le m$

$\frac{1}{mn} \le x \le \frac{m}{n}$

$\frac{m}{n} - \frac{1}{mn} = \frac{1}{2013}$

$n = 2013m - \frac{2013}{m}$

For n to be an integer, m must divide 2013, and m > 1. To minimize n, m should be as small as possible because increasing m will decrease $\frac{2013}{m}$ , the amount you are subtracting, and increase 2013m , the amount you are adding; this also leads to a small m which clearly minimizes m + n.

We let m equal 3, the smallest non-1 factor of 2013. $n = 2013*3 - \frac{2013}{3} = 6039 - 671 = 5368$

$m + n = 5371,$ so the answer is $\boxed{371}$